2021 AMC 10B Problems/Problem 1

Revision as of 17:36, 11 February 2021 by Smarty101 (talk | contribs) (Solution)

How many integer values of $x$ satisfy $|x|<3\pi$?

$\textbf{(A)} ~9 \qquad\textbf{(B)} ~10 \qquad\textbf{(C)} ~18 \qquad\textbf{(D)} ~19 \qquad\textbf{(E)} ~20$

Solution by smarty101

Since $3\pi$ is about $9.42$, we multiply 9 by 2 and add 1 to get $\boxed{\textbf{(D)}\ ~19}$